Đến nội dung

superpower nội dung

Có 486 mục bởi superpower (Tìm giới hạn từ 07-05-2020)



Sắp theo                Sắp xếp  

#653626 CMR: $\frac{1}{a+b}+\frac{1}...

Đã gửi bởi superpower on 10-09-2016 - 21:13 trong Bất đẳng thức và cực trị

Tại sao đoạn Cauchy_Schwwarz lại ra như thế kia vậy a

E nhớ là BĐT Cauchy-schwwarz đâu có như vậy

Với cả dấu bằng của a, em nghĩ có vấn đề, áp dụng C-S ==> a=b=c ==> ko t/m

Đó là C-S dạng engel đó bạn

$\sum \frac{c}{a+b} = \sum \frac{c^2}{ca+cb} \geq \frac{(a+b+c)^2}{2(ab+bc+ca)} $

Về chỗ dấu bằng

Thì theo bđt C-S thì xảy ra khi các biến tỉ lệ với quy ước mẫu bằng 0 thì tử bằng 0

Do đó vẫn đảm bảo dấu bằng tại biên của bài toán




#653612 $A=\sum x^{3} +8(xy^{2}+yz^{2}+zx^...

Đã gửi bởi superpower on 10-09-2016 - 19:29 trong Bất đẳng thức - Cực trị

Cho $x,y,z\geq 0;x+y+z=4.$

Tìm max: $A=\sum x^{3} +8(xy^{2}+yz^{2}+zx^{2})$

Bài này mình mò được điểm rơi là $\left ( 0;2;2 \right )$ mà chưa tìm được cách.

Đặt $f(x,y,z) = \sum x^3 + 8(xy^2+yz^2+zx^2) $

Ta có $f(x+z,y+z,0) = (x+z)^3 + (y+z)^3 + 8(x+z)(y+z)^2 $

Ta chứng minh $f(x,y,z) \leq f(x+z,y+z,0 ) $

Là hiển nhiên

Do đó, ta càn tìm max của

$B= a^3+b^3+8.ab^2 $ với $a+b \leq 4 $

Tới đây dễ rồi




#653312 Đề thi chọn đội tuyển chuyên Thái Bình

Đã gửi bởi superpower on 08-09-2016 - 15:23 trong Thi HSG cấp Tỉnh, Thành phố. Olympic 30-4. Đề thi và kiểm tra đội tuyển các cấp.

Bài 2: (4 điểm)
Cho dãy số ${u_n}$ xác định bởi $u_0=1; u_1=2$
$$u_{n+2}=4u_{n+1}-u_n$$
Tìm cồng thức tổng quát của dãy. Xác định tất cả các giá trị của $n$ để $u_n-1$ là số chính phương.
 

Ta có công thức tổng quát của dãy là

$u_n= \frac{1}{2} )(2 + \sqrt{3})^n + \frac{1}{2}(2-\sqrt{3})^n $ 

Ta sẽ chứng minh $n$ lẻ thỏa YCBT

Thật vậy, ta có 

$(2+\sqrt{3})^n +(2-\sqrt{3})^n -2 = 2k^2$

Xét $n$ lẻ thì ta sẽ chọn $k = (\frac{(\sqrt{3}+1)^n}{2^{\frac{n+1}{2}}} )^2 - (\frac{(\sqrt{3}-1)^n}{2^{\frac{n+1}{2}}} )^2 $

Dễ chứng minh $k$ nguyên 

Do đó ta có đpcm 

Còn với $n$ chẵn thì ta có pt lúc sau sẽ có dạng $3x^2=2k^2 $

Dễ thấy cái này chỉ có nghiệm $(0,0)$

Do đó $n=0 $ và $n $ lẻ thỏa YCBT




#653256 $I$ là tâm đường tròn nội tiếp tam giác $ABE$.

Đã gửi bởi superpower on 07-09-2016 - 22:59 trong Hình học

Bài toán: Cho tam giác $ABC$ cân tại $A$ có $H,M$ lần lượt là trung điểm của $BC,AC$. Đường tròn ngoại tiếp tam giác $BCM$ cắt đoạn $AH$ tại $D$ và đường tròn ngoại tiếp tam giác $ABD$ cắt đoạn $BM$ tại $K$. Gọi $I$ là giao điểm của $AK$ và $BD$, $E$ là giao điểm của $CI$ với $BM$. Chứng minh:
1.Tam giác $AKC$ vuông.
2.$I$ là tâm đường tròn nội tiếp tam giác $ABE$.

Bài hình đầu tiên mình giải trên VMF :))

Ta chứng minh  $\triangle AKC $ vuông như sau

$\widehat {KAM} =\widehat {BHD}=\dfrac{1}{2} \widehat{BDC} = \dfrac{1}{2} \widehat{BMC}  $

Do đó $MK=MA $

Suy ra $\triangle AKC$ vuông

b/  Ta có $IK.ID=ID.IB => I $ thuộc trục đẳng phương của $(BMC)$ và $(AC) $

Do đó $CI $ cắt $(BMC)$ tại $J $

Mặt khác, ta cũng có $\widehat {MBC} = \widehat{MJC} = \widehat{MCJ} $

Do đó $MA^2=MC^2=ME.MB $

Từ đó biến đổi góc thêm 1 tí là ra

Hình gửi kèm

  • Zalo_ScreenShot_7_9_2016_225214.png



#652935 tìm tất cả các stn n sao cho n chia hết cho [$\sqrt{n}...

Đã gửi bởi superpower on 05-09-2016 - 20:20 trong Số học

tìm tất cả các stn n sao cho n chia hết cho [$\sqrt{n}$]

Đặt $[\sqrt{n} ] = d $

Khi đó $d \leq \sqrt{n} < d+1 $

$d^2 \leq n < d^2+ 2d $

Do đó $n= d^2 + r$

Với $r= 1,2,...,2d-1 $

Mặt khác $n \vdots d => r \vdots d $

Do đó, $r=d $

Vậy $n= d^2+ d $

Thử lại thỏa 




#652777 Đề chọn đội tuyển Quốc Gia môn Toán

Đã gửi bởi superpower on 04-09-2016 - 17:28 trong Thi HSG cấp Tỉnh, Thành phố. Olympic 30-4. Đề thi và kiểm tra đội tuyển các cấp.

 

Bài toán 3: Cho $P(n)$ là một đa thức( hệ số thực) của biến tự nhiên $n$ thỏa mãn:

$P(n)=1^{2003}+2^{2003}+...+n^{2003},\forall n\in N^{*}$.

Chứng minh rằng: đa thức $P(n)$ chia hết cho đa thức $Q(n)=n^2(n+1)^2$

 

 

Bài 3 có sai gì không bạn

Dễ thấy $P(1) = 1$ không chia hết cho $Q(1) $

$P(2) $ cũng vậy 




#652776 $x_{n+1} = \dfrac{x_n}{2} + \dfrac{n^2}{4n^2+a}.\sqrt{x_n...

Đã gửi bởi superpower on 04-09-2016 - 17:25 trong Dãy số - Giới hạn

Full cho em hộ anh. Em mới tìm hiểu phần này thôi =))

Thật ra bài này em có thể đọc trong phần lời giải và bình luận kì thi VMO 2015 của thầy Dũng

Có 2 cách giải

Anh có 1 lời giải khác, dùng bổ đề, nhưng chứng minh tương tự thôi nên không đăng

File gửi kèm




#652767 $x_{n+1} = \dfrac{x_n}{2} + \dfrac{n^2}{4n^2+a}.\sqrt{x_n...

Đã gửi bởi superpower on 04-09-2016 - 16:20 trong Dãy số - Giới hạn

a) Ta chứng minh dãy số luôn dương và là dãy giảm bằng Quy nạp.

Dãy bị chặn dưới bởi $0$, Theo Weierstrss thì dãy có giới hạn. Đặt: $lim(x_{n})=\alpha$.

Suy ra ta giải phương trình: $\alpha =\frac{\alpha }{2}+\frac{1}{4}\sqrt{\alpha ^2+3}\Rightarrow \alpha =1$ do $\alpha >0$.

b) Với $a=1$ thì :

$1-\frac{2}{n+1}\leq x_{n+1}\leq \frac{x_n}{2}+\frac{1}{4}{\sqrt{x_{n}^2+3}}$

Giới hạn của 2 dãy đều bằng $1$ nên Theo nguyên lý kẹp ta có  $lim(x_{n})=1$.

Bạn chứng minh ý đầu chưa

Đó mới là ý chính của bài toán

Ngoài ra ở ý $b$ ngoài cách đó

Thì mình còn 1 cách chứng minh là khi $a=1 $ có $lim =1 $ luôn




#652278 $\frac{a}{a^{2}+3}+\frac{b...

Đã gửi bởi superpower on 01-09-2016 - 20:45 trong Bất đẳng thức và cực trị

Cho các số dương a,b,c thỏa $abc=1$ .Chứng minh rằng:

 

$\frac{a}{a^{2}+3}+\frac{b}{b^{2}+3}+\frac{c}{c^{2}+3}\leq \frac{3}{4}$

Xét hàm số $f(x) = \frac{x}{x^2+3} - \frac{1}{8}lna $ trên $(0,+\infty )$ 

Có $f'(x) =\frac{3-x^2}{(x^2+3)^2} - \frac{1}{8x} $

Kẻ bảng biến thiên, ta chứng minh đc $f(x) \leq f(1) $

Do đó cộng hết lại ta có đpcm 




#651983 CMR: $\sum\frac{a}{b+c}+\frac{ab...

Đã gửi bởi superpower on 30-08-2016 - 18:13 trong Bất đẳng thức và cực trị

Ngay từ đầu bài toán em chuẩn hóa luôn a+b+c=3 thì có đc ko a?

đc em. Những sẽ không quy đc về soS




#651734 $\frac{8(a^3+b^3+c^3)}{(a+b)(b+c)(a+c)} \g...

Đã gửi bởi superpower on 28-08-2016 - 22:51 trong Bất đẳng thức và cực trị

Cho a,b,c > 0.CMR:

$\frac{8(a^3+b^3+c^3)}{(a+b)(b+c)(a+c)} \geq \frac{4(a^2+b^2+c^2)}{ab+bc+ca} + 1$

$pqr$ thôi

Chú ý, khi chuyển $pqr$, nhờ tính đơn điệu của $r$, ta có thể đưa về chứng minh bđt 1 biến




#651688 CMR: $\sum\frac{a}{b+c}+\frac{ab...

Đã gửi bởi superpower on 28-08-2016 - 19:54 trong Bất đẳng thức và cực trị

Anh có thể giúp em viết rõ phần màu xanh này đc ko ạ, em thấy hơi khó hiểu ~O)  

2a+b+c > a+b+c vì a,b,c >0, mới cả anh viết hộ em dấu bằng xảy ra khi nào ạ?

 

BĐT $<=> \sum \frac{a}{b+c} -\frac{3}{2} + \frac{abc}{2(a^3+b^3+c^3} - \frac{1}{6} \geq 0 $

        $<=> \sum (a-b)^2 . \frac{1}{2(a+c)(b+c)} - \sum (a-b)^2. \frac{a+b+c}{12(a^3+b^3+c^3) } \geq 0 $

        $<=> \sum (a-b)^2 . (\frac{1}{2(a+c)(b+c)} -\frac{a+b+c}{12(a^3+b^3+c^3) } ) \geq 0 $

Giả sử $a \geq b \geq c $

Suy ra đc $S_c \leq S_b \leq S_a $

Do đó, ta cần chứng minh 

$S_b + S_c >0 <=> 3(2a+b+c)(a^3+b^3+c^3) \geq (a+b+c)(a+b)(b+c)(c+a) $

Do tính thuần nhất, ta có thể chuẩn hóa $a+b+c =3 $

Và chú ý rằng $2a+b+c \geq a+b+c $

Chuyển $pqr $

Do đó, ta chỉ cần chứng minh $3(p^3-3pq+3r) \geq pq -r $

$<=> r \geq \frac{30q-81}{10} $

Nếu $q \leq \frac{27}{10} $ ta có đpcm 

Nếu $3 \geq q \geq \frac{27}{10} $, áp dúng bdt Schur bậc 3, ta cần chứng minh 

$\frac{12q-27}{9} \geq \frac{30q-81}{10} <=> 150q \leq 459$ đúng

Vậy ta có đpcm 

Bây giờ, đẳng thức xảy ra khi $a=b=c$

Vì bên ngoài các đại lượng $S_a, S_b , S_c $ đã có $(a-b)^2, (b-c)^2, (c-a)^2 $ rồi

Mặt khác, theo tiêu chuẩn chứng minh SOS cho $a \geq b \geq c $

Là $S_a(b-c)^2 + S_b(a-c)^2  +S_c( a-b)^2 \geq 0 $

Nếu có $S_b \geq 0 , S_b + S_c \geq 0 , S_b+ S_a \geq 0 $ là ta có dương

Cái trên anh áp dụng luôn

Do có $S_c \leq S_b \leq S_a $

nên chỉ cần chứng minh $S_b + S_c >0 $ thôi

Khi đó anh sẽ thử chứng minh 

$3(2a+b+c)(a^3+b^3+c^3) \geq (a+b+c)(a+b)(b+c)(c+a) $

Mà đã có $2a+b+c \geq a+b+c $

Nên chỉ cần chứng minh $3(a^3+b^3+c^3) \geq (a+b)(b+c)(c+a) $ thôi

Mà cái này anh quy về $pqr$ để chứng minh thì thấy nó đúng




#651675 Chứng minh giá trị của tổng là e

Đã gửi bởi superpower on 28-08-2016 - 19:02 trong Dãy số - Giới hạn

Chứng minh rằng dãy vô hạn $\frac{1}{0!}+\frac{1}{1!}+\frac{1}{2!}+\frac{1}{3!}+....$ có giá trị là e

Xét khai triển taylor của $e^x $

$e^x = 1+ x+ \frac{x^2}{2!} + \frac{x^3}{3!} + ... $

Cho $x=1$, ta tính đc

$e  =  1+ 1 + \frac{1}{2!} + \frac{1}{3!} + ... $

Do đó , ta có đpcm 




#651652 Chứng minh rằng: $x^{3}+y^{3}\geq x^{2...

Đã gửi bởi superpower on 28-08-2016 - 15:52 trong Bất đẳng thức và cực trị

Chứng minh rằng: $x^{3}+y^{3}\geq x^{2}y+xy^{2}, x,y\geq 0$

Cách khác

Ta có 

$x^3-x^2y + y^3 - xy^2 = x^2(x-y) + y^2(y-x) = (x-y)(x^2-y^2)=  (x-y)^2(x+y) \geq 0 $




#651386 Chứng minh rằng: $a+b+c \leq 1+abc$

Đã gửi bởi superpower on 26-08-2016 - 20:30 trong Bất đẳng thức - Cực trị

Cho $a,b,c$ là các số thực dương thỏa mãn $(a+1)(b+1)(c+1)=1+4abc$

Chứng minh rằng: $a+b+c \leq 1+abc$

Thay $(a;b;c) \rightarrow (\dfrac{1}{a};\dfrac{1}{b};\dfrac{1}{c})$, ta được bài toán mới

Ta quy về pqr, bđt cần chứng minh trở thành

Cho $p+q=3$

a/ Chứng minh $q < \ 1 +r $

Ta cần chứng minh $r > 2 -p $

Theo bđt Schur bậc 3, ta $p^3 -4pq +9r \geq 0 <=> r \geq \dfrac{12p-4p^2-p^3}{9} $

Ta cần chứng minh $\dfrac{12p-4p^2-p^3}{9} \geq 2-p <=> p^3+4p^2-21p+18 <0<=> 1,24 <p<2 $

Vậy bđt đúng khi $1,24<p<2 $

Mặt khác, ta $3=p+q \leq p+\dfrac{p^2}{3} <=> p \geq 1,8 $

Nếu $p \geq 2 $thì $2-p \leq 0 \leq r $

Do đó bài toán được chứng minh




#651380 CMR: $\sum\frac{a}{b+c}+\frac{ab...

Đã gửi bởi superpower on 26-08-2016 - 20:07 trong Bất đẳng thức và cực trị

Cho a,b,c > 0. CMR:

$\frac{a}{b+c}+\frac{b}{a+c}+\frac{c}{a+b}+\frac{abc}{2(a^3+b^3+c^3)} \geq \frac{5}{3}$

BĐT $<=> \sum \frac{a}{b+c} -\frac{3}{2} + \frac{abc}{2(a^3+b^3+c^3} - \frac{1}{6} \geq 0 $

        $<=> \sum (a-b)^2 . \frac{1}{2(a+c)(b+c)} - \sum (a-b)^2. \frac{a+b+c}{12(a^3+b^3+c^3) } \geq 0 $

        $<=> \sum (a-b)^2 . (\frac{1}{2(a+c)(b+c)} -\frac{a+b+c}{12(a^3+b^3+c^3) } ) \geq 0 $

Giả sử $a \geq b \geq c $

Suy ra đc $S_c \leq S_b \leq S_a $

Do đó, ta cần chứng minh 

$S_b + S_c >0 <=> 3(2a+b+c)(a^3+b^3+c^3) \geq (a+b+c)(a+b)(b+c)(c+a) $

Do tính thuần nhất, ta có thể chuẩn hóa $a+b+c =3 $

Và chú ý rằng $2a+b+c \geq a+b+c $

Chuyển $pqr $

Do đó, ta chỉ cần chứng minh $3(p^3-3pq+3r) \geq pq -r $

$<=> r \geq \frac{30q-81}{10} $

Nếu $q \leq \frac{27}{10} $ ta có đpcm 

Nếu $3 \geq q \geq \frac{27}{10} $, áp dúng bdt Schur bậc 3, ta cần chứng minh 

$\frac{12q-27}{9} \geq \frac{30q-81}{10} <=> 150q \leq 459$ đúng

Vậy ta có đpcm 




#651215 Giải hệ phương trình bậc cao.

Đã gửi bởi superpower on 25-08-2016 - 15:51 trong Phương trình - Hệ phương trình - Bất phương trình

Giải hệ phương trình sau: $\left\{\begin{matrix} x^{5}+xy^{4}=y^{10}+y^{6} & & \\ \sqrt{4x+5}+\sqrt{y^{2}+8}=6 & & \end{matrix}\right.$

Đặt $t=y^2 $

Từ pt $(1)$ Suy ra

$x^5-t^5 = t^2(t-x) <=> (x-t)(x^4+x^3t+x^2t^2+xt^3+t^4+t^2 ) =0 $

Mà ta có $x^4+x^3t+x^2t^2+xt^3+t^4+t^2 = (x+t)(x^3+t^3) + x^2t^2+t^2 = (x+t)^2(x^2-xt+t^2) + x^2t^2+t^2 >0 $

Do đó $x=t $

Hay $x=y^2 \geq 0 $

Thay vào pt dưới ta được

$\sqrt{4x+5} +\sqrt{x+8} = 6 $

Nếu $x > 1 $ thì $VT > 6 $ vô lí

Nếu $x < 1 $ thì $VP < 6 $ vô lí

Do đó $x=1 $ là nghiệm duy nhất

Khi đó $y=1 ; y=-1 $




#651214 Đề chọn đội tuyển Ams 2016-2017

Đã gửi bởi superpower on 25-08-2016 - 15:41 trong Thi HSG cấp Tỉnh, Thành phố. Olympic 30-4. Đề thi và kiểm tra đội tuyển các cấp.

Xét $f(x) = \frac{3x-1}{x} $

Có $f'(x) = \frac{1}{x^2 } >0 $

Mà mặt khác ta có $x_2 < x_1 => x_n giảm $

Mặt khác, ta có $x_1 \geq \frac{3+\sqrt{5}}{2} $

Giả sử $x_n \geq \frac{3+\sqrt{5}}{2} $ đúng với $n$, ta chứng minh đúng với $n+1 $

Tức là chứng minh 

$\frac{3x_n-1}{x_n} \geq \frac{3+\sqrt{5}}{2} <=> x_n \geq \frac{3+\sqrt{5}}{2} $

Do đó ta có $x_n$ giảm, bị chặn dưới bởi $\frac{3+\sqrt{5}}{2} $ do đó, tồn tại $L$ bằng $lim x_n $

Mà khi chuyển sang giới hạn, ta tính đc $L=\frac{3+\sqrt{5}}{2} $

Mặt khác $\frac{y_{n+1}}{y_n } = \frac{3+\sqrt{5}}{2x_{n+1}}  \leq 1$

Do đó $y_n $ giảm, bị chặn dưới bởi $0$ 

Suy ra tồn tại $L'= lim y_n $

Thay vô $L'=\frac{3+\sqrt{5}}{2L} . L' => L'=0 $




#651093 Đề kiểm tra đội tuyển toán Chuyên Bảo Lộc (Lâm Đồng)

Đã gửi bởi superpower on 24-08-2016 - 17:18 trong Thi HSG cấp Tỉnh, Thành phố. Olympic 30-4. Đề thi và kiểm tra đội tuyển các cấp.

LẦN III

Câu 1 (3 điểm): Cho dãy số thực $\left ( u_{n} \right )$ được xác định bởi:

$$\left\{\begin{matrix} u_{1}=2014 & & \\ u_{n+1}=\dfrac{u_{n}^{4}+2013^{2}}{u_{n}^{3}-u_{n}+4026},\,\,\,\, n\in \mathbb{N^{*}} & & \end{matrix}\right.$$

Đặt

$$v_{n}=\sum_{k=1}^{n}\dfrac{1}{u_{k}^{2}+2013},\,\,\,\, \forall n\in \mathbb{N^{*}}$$

Tính $\lim v_{n}$.

Biến đổi tươơng đương sơ sơ, ta được 

$\frac{1}{u_n^2 +2013} = \frac{1}{u_n^2-2013} - \frac{1}{u_{n+1}^2 -2013} $

Tới đây cộng lại là dễ rồi




#650991 Đề chọn đội tuyển chuyên Nguyễn DU- Đăklak 2016-2017

Đã gửi bởi superpower on 23-08-2016 - 21:18 trong Thi HSG cấp Tỉnh, Thành phố. Olympic 30-4. Đề thi và kiểm tra đội tuyển các cấp.

Bài 4:

a/ Câu bđt  Cho $a,b,c >0 $ thỏa $abc=1 $

Chứng minh $\sum \frac{1}{a+2} \leq 1 $

Quy đồng hết lên, ta được

$\sum (2+a)(2+b) \leq (2+a)(2+b)(2+c) $

$<=> 12+ 4\sum a+ \sum ab \leq 8+4\sum a + 2\sum ab + abc $

$<=> \sum ab + abc \geq 4 $

Ta có $ab+bc+ca \geq 3\sqrt[3]{a^2b^2c^2} =1 $

Do đó ta có đpcm 

b/ Câu hàm $f(5x+y) = f(x)+f(2y) + 4x-y $

Thay $x=y=0 => f(0)=0 $

Thay $x=0 => f(y) = f(2y)-y <=> f(y)-y = f(2y) -2y $

Đặt $g(x)= f(x) -x $

Do đó $g(x)= g(2x) $

Do $f(x)$ liên tục nên $g(x)$ liên tục

Do đó $g(x) = g(\frac{x}{2}) =...= g(\frac{x}{2^n} )$

Với mỗi $x$ cố định cho $n -> +\infty=> g(x) = g(0) =0$

Khi đó $f(x) = x $

Thử lại thỏa 

 

 

 

 




#650826 CMR:$\sum \frac{a^{2}}{b^{2...

Đã gửi bởi superpower on 22-08-2016 - 19:24 trong Bất đẳng thức và cực trị

Cho a,b,c >0 CMR:$\sum \frac{a^{2}}{b^{2}+c^{2}}\geq \sum \frac{a}{b+c}$

Ta có thể chứng minh hàm số

$f(x) = \sum \frac{a^x}{b^x+c^x} $ đồng biến trên $R$




#650697 Chứng minh rằng $\sqrt{a^{2}+bc}+\sqrt...

Đã gửi bởi superpower on 21-08-2016 - 21:01 trong Bất đẳng thức và cực trị

Cho a, b, c là các số thực không âm thỏa mãn a+b+c=2 . Chứng minh rằng $\sqrt{a^{2}+bc}+\sqrt{b^{2}+ca}+\sqrt{c^{2}+ab}\leq 3$

Bài này dồn biến thôi

Đặt $f(a,b,c) = \sum \sqrt{a^2+bc} $

Giả sử $a \geq b \geq c $

Khi đó $f(a+\frac{c}{2} ,b+\frac{c}{2} ; 0 ) = a+\frac{c}{2} + b+\frac{c}{2} + \sqrt{(a+\frac{c}{2})(b+\frac{c}{2} ) } $

Ta cần chứng minh $f(a+\frac{c}{2}; b+\frac{c}{2} ; 0 ) \geq f(a,b,c) $

Lấy 2 cái trừ nhau và nhân liên hợp, ta cần chứng minh 

$c[\frac{a-b+\frac{c}{4} }{a+\frac{c}{2} + \sqrt{a^2+bc} } + \frac{b-a+\frac{c}{4} }{b+\frac{c}{2} + \sqrt{b^2+ac} }  + \frac{\frac{a+b}{2} - \frac{3c}{4} }{ \sqrt{(a+\frac{c}{2})(b+\frac{c}{2} ) }+\sqrt{c^2-ab} }] \geq 0$

Do $c \geq 0 $ và $\frac{a+b}{2} - \frac{3c}{4} \geq 0 $

Do đó ,ta cần chứng minh tổng 2 căn đầu $>0 $

Thật vậy, ta có $a+\frac{c}{2} + \sqrt{a^2+bc} \geq b+\frac{c}{2} + \sqrt{b^2+ac} $

Do đó, ta có

$\frac{a-b+\frac{c}{4} }{a+\frac{c}{2} + \sqrt{a^2+bc} } + \frac{b-a+\frac{c}{4} }{b+\frac{c}{2} + \sqrt{b^2+ac} } $

$\geq \frac{a-b+\frac{c}{4} }{a+\frac{c}{2} + \sqrt{a^2+bc} } + \frac{b-a+\frac{c}{4} }{a+\frac{c}{2} + \sqrt{a^2+bc} } = \frac{c}{2(a+\frac{c}{2} + \sqrt{a^2+bc})} >0  $

Do đó $f(a+\frac{c}{2}; b+\frac{c}{2} ; 0 ) \geq f(a,b,c) $

Đặt $a+\frac{c}{2} =x ; b+\frac{c}{2} = y $

Khi đó $x+y = 2 $

Ta cần chứng minh $x+y+\sqrt{xy} \leq 3 $

Mà này hiển nhiên do $\sqrt{xy} \leq \frac{x+y}{2} $ 

Do đó ta có đpcm

Dấu bằng xảy ra khi $a=b=1; c=0 $ và các hoán vị tương ứng




#650180 Topic: [LTDH] Mỗi ngày hai bất đẳng thức.

Đã gửi bởi superpower on 18-08-2016 - 08:18 trong Bất đẳng thức và cực trị

Lời giải bài 21 và bài 22 các bạn VODANH9X và phamngochung9a đã giải chuẩn rồi, mình xin đề xuất hai bài tiếp theo:

Bài 23: Cho các số thực $a,b,c\in [1,2]$. Tìm GTNN của biểu thức:

$P=\frac{(a+b)^2}{c^2+4(ab+bc+ca)}$

 

Xét hàm số $f(c) = c^2 + 4(ab+bc+ca) $ là hàm đồng biến theo biến $c$

Do đó để $P_{min} $ thì $c $ max

Do đó. ta cần tìm min của biểu thức

$P=\frac{(a+b)^2}{4+4(ab+2a+2b) } $

Mặt khác

Xét hàm số $f(a) = \frac{a^2+2ab+b^2}{(4b+8)a + 4+8b } $

Có $f'(a) = \frac{(4b+8)a^2+ (16b+8)a -4b^3 +8b^2+8b }{[(4b+8)a+4+8b]^2}$

Mà ta có $-4b^3+8b^2+8b > 0 $ (do $b \in [1;2] $)

Do đó $f'(a) >0 $

Tương tự dễ chứng minh $f'(b) >0 $

Mà $a,b \geq 1 => f(a,b) \geq f(1,1) = \frac{1}{6} $

Dấu bằng xảy ra khi $a=b=1; c=2 $ 




#650178 Anh Huyện Cho E hỏi

Đã gửi bởi superpower on 18-08-2016 - 08:07 trong Bất đẳng thức và cực trị

Trong cuốn sách của anh có viết như thế này . Vậy cho e hỏi sao lại đưa được về thế này ạ

80mF4z9.png

Cái đó là dạng tương đương mà bạn

Ảnh viết vậy chỉ để dễ phân tích SS thôi




#649774 $p^3-2p^2+p+1=3^n$

Đã gửi bởi superpower on 15-08-2016 - 19:03 trong Số học

Tìm số nguyên dương $n$ và số nguyên tố $p$ thỏa 

$p^3-2p^2+p+1=3^n$

Với $p=2 => n=1 $

Phân tích như sau

Do dễ thấy $n$ chẵn nên đặt $n=2k $

$p(p-1)^2  =(3^k-1)(3^k+1 ) $

TH1: $3^k -1 \vdots p => 3^k -1 = pr $

Thay vào lại đưa về PT bậc 2 theo $p: p^2-(2+r^2)pp+1-2r = 0 $

Có $\triangle = r^4+4r^2+8r$

Mà ta chứng minh được $(r^2+3)^2 > \triangle > (r^2+2)^2 $

Do đó vô lí

TH2: $3^k +1 \vdots p $

Làm tương tự tính được $\triangle=r^4+4r^2- 8r $

Ta có $r=1 $ không thoả nên  $r \geq 2 $

Khi đó $(r^2+2)^2 > \triangle \geq r^4$

Do đó, giải ra $r=2 => p =5 $